Тёмный

a nice functional equation. 

Michael Penn
Подписаться 300 тыс.
Просмотров 28 тыс.
50% 1

🌟Support the channel🌟
Patreon: / michaelpennmath
Merch: teespring.com/stores/michael-...
My amazon shop: www.amazon.com/shop/michaelpenn
🟢 Discord: / discord
🌟my other channels🌟
Course videos: / @mathmajor
non-math podcast: / @thepennpavpodcast7878
🌟My Links🌟
Personal Website: www.michael-penn.net
Instagram: / melp2718
Randolph College Math: www.randolphcollege.edu/mathem...
Research Gate profile: www.researchgate.net/profile/...
Google Scholar profile: scholar.google.com/citations?...
🌟Suggest a problem🌟
forms.gle/ea7Pw7HcKePGB4my5

Опубликовано:

 

2 мар 2022

Поделиться:

Ссылка:

Скачать:

Готовим ссылку...

Добавить в:

Мой плейлист
Посмотреть позже
Комментарии : 103   
@gilmonat3155
@gilmonat3155 2 года назад
17:49 i think there's a mistake. Equating the arguments gives f(n)+1 = n+1 =>f(n)=n , but not f(n+1)=f(n)+1 like shown on the board. Correct reasult either way :)
@somasahu1234
@somasahu1234 2 года назад
Lol, yeah!
@amirb715
@amirb715 2 года назад
LOL...yeah obviously....big error although the final answer came out correct
@tordjarv3802
@tordjarv3802 2 года назад
That was what I thought as well.
@calde607
@calde607 2 года назад
yeah lol
@wesleydeng71
@wesleydeng71 2 года назад
A lucky error indeed!
@unflexian
@unflexian 2 года назад
I think a good video topic would be Cauchy's functional equation. It's the equation f(x+y)=f(x)+f(y), and you could show the proof that solutions exist not of the form f(x)=ax, and that all such solutions are dense, i.e the function passed through all discs on the plane no matter how small.
@swenji9113
@swenji9113 2 года назад
the existence of such solutions is not trivial at all tho, it requires the use of axiom of choice and knowledge about its various equivalent forms.. how would you approach this?
@DeanCalhoun
@DeanCalhoun 2 года назад
at the point that we got f(m^2)=mf(m) I guess the solution would be the identity, though I wouldn’t have been able to show it. great video!
@cosimodamianotavoletti3513
@cosimodamianotavoletti3513 2 года назад
at 16:03 we apply periodicity, but we have only proved that f(x+b-a)=f(x) if x>=a. We can prove pretty easily that y=1: f(y^2)=yf(y)
@jordanweir7187
@jordanweir7187 2 года назад
This is so brilliantly instructive, nice explanation
@themathsgeek8528
@themathsgeek8528 2 года назад
Great video, prof Penn:) Thanks!
@kimbel2804
@kimbel2804 2 года назад
A little faster way is to prove injectivity is to realise that when holding f(m+n) constant (e.g. choosing m->m+x and n->n-x) and changing x, we get arbitrarily large image of f (when n is chosen carefully), which proves that f is clearly not periodic.
@pablomartinsantamaria8689
@pablomartinsantamaria8689 2 года назад
I was thinking about just saying that if there's a period, then there would be infinite z s.t. f(z)=0, which contradicts our assumption from case 2
@andrewulinov3920
@andrewulinov3920 2 года назад
Well, you are not able to do this. When changing x your new variable (n := n-x) becomes negative, and we are working in nonnegative numbers.
@kimbel2804
@kimbel2804 2 года назад
@@andrewulinov3920 That is why we need to select n to be sufficiently large and then intuitively when we let n approach infinity, we get infinitely large image of f.
@chausolaris
@chausolaris 2 года назад
For case 2, we could set m=-n, then f(n^2-nf(n))=0, by case 2 assumption, n^2-nf(n)=0 => f(n)=n
@janekgroe4304
@janekgroe4304 2 года назад
Nah we're only talking about numbers bigger than/equal to zero so you can't just take -n. Also you're using injectivity in your argumentation which means that it doesn't make the proof any shorter since showing injectivity was the only thing taking a lot of time
@andrycraft69
@andrycraft69 2 года назад
@@janekgroe4304 You're right about the first thing, but I think he wasn't using injectivity. He argued that, since the value of the function is zero, by the assumption that we made in the second case, the only possible way to get zero is for the argument of the function to be zero.
@janekgroe4304
@janekgroe4304 2 года назад
@@andrycraft69 oh yes right I forgot we assumed "injectivity at 0"
@littlekeegs8805
@littlekeegs8805 2 года назад
At 17:50, I believe you meant to write that 1 + f(n) = n + 1, since that's the interior of those two functions. From that, it's immediately obvious that f(n) = n.
@ojasdeshpande7296
@ojasdeshpande7296 2 года назад
Wrong step again leads to correct solution😂
@samuelbosman9572
@samuelbosman9572 2 года назад
This video was a whole lot of magic for me. Lol. Cool result!
@HagenvonEitzen
@HagenvonEitzen 2 года назад
09:00 Use twice the period, i.e., f(y + 2z) = f(y+z) = f(z), leading to (y+2z-1)f(y) 0, so f(y)=0.
@giorgioleoni3471
@giorgioleoni3471 2 месяца назад
At 7:30, wouldn't it be easier to write f(y) >= f(y2)=yf(y)>f(y) unless y=0?
@manucitomx
@manucitomx 2 года назад
Thank you, professor.
@ty6339
@ty6339 2 года назад
16: 10 How do we know that f(y) = f(y + b - a), we have the periodicity only for y => a, right?
@mathflipped
@mathflipped 2 года назад
This is a tough one. Well done, Michael!
@Packerfan130
@Packerfan130 Год назад
You went from f(1 + f(n)) = f(n + 1) to f(n + 1) = f(n) + 1 quoting injectivity. But wouldn't injectivity yield, f(n) + 1 = n + 1, then immediately follows that f(n) = n. If f(a) = f(b), then a = b. Here a = f(n) + 1 and b = n + 1.
@romajimamulo
@romajimamulo 2 года назад
Now what if it was over the rationals or the reals?
@almafater
@almafater 2 года назад
Functional equations are cool but it’s somewhat disappointing that we almost always end up with something like f(x)=x 😅 Nice video though!
@advaykumar9726
@advaykumar9726 2 года назад
The identity function
@igorvereshch944
@igorvereshch944 2 года назад
you can actually get the answer from f(m^2)=mf(m) by assuming f(m)=sum of i from 1 to inf a(i)*m^i (no need for 0-th term because f(0)=0). S is sum from 1 to inf. then we get S(a(2i)*m^2i) = S(a(i)*m^(i+1)) First term is a(1)*m^2 for both, plus all a(even) are 0 because they are coefs for m^odd which exists only on the right side and it has to work for any natural m. By getting rid of all of that, Sum from 2 to inf (a(i)-a(2i-1))*m^2i = 0 so every single a(i)-a(2i-1) is 0. For any odd value of i we get a(i)=a((i+1)/2) but we'll eventually get to a(even) on the right side, so all values of a except a(1) are 0. Thus f(m)=a*m. You can find a being either 1 or 0 by putting f(m) into initial equation.
@rohzzup
@rohzzup 2 года назад
Bonus points for using correct Indian map.
@jayantanayak4981
@jayantanayak4981 2 года назад
You're soooooooooo underrated!!!
@noumanegaou3227
@noumanegaou3227 2 года назад
You have a mistake If f injective And for all n in N f(1+f(n))=f(n+1) Then 1+f(n)=n+1 Then f(n) =n
@joehead4081
@joehead4081 2 года назад
In the second case, how can f(n) have the behavior of "periodic after a point" if it's injective?
@ojasdeshpande7296
@ojasdeshpande7296 2 года назад
That's what...the period was b-a which is 0. This by definition of periodicity, f(x) = f(0+x) which is an identity.
@matanah1989
@matanah1989 2 года назад
a little nagging regarding the proof using [b-a] periodicity, if you ends up with b-a=0 then there is no periodicity, the image is not necessarily finite and does not necessarily have a maximum (as in the final results). a more rigorous way is to say, assuming b-a>0 there is periodicity... concluding b-a must be 0, in contradiction
@ezequielangelucci1263
@ezequielangelucci1263 6 месяцев назад
is the same, b-a=0 means no periodicity but also means injectivity is true
@wesleydeng71
@wesleydeng71 9 месяцев назад
It is impossible for f(x) to be periodic because f(m^2)=mf(m) can be arbitrarily large unless f(x) is always 0. This would save a lot of time.😂
@stefanschroder4694
@stefanschroder4694 2 года назад
Nice🙌
@gamerpedia1535
@gamerpedia1535 9 месяцев назад
Here was my process f(m²+mf(n)) = mf(m+n) When m = 0 we get f(0) = 0 When n = 0 we get f(m²) = mf(m) m = 1 gives us f(1+f(n)) = f(n+1) Applying the inverse function on both sides gives us f(n)+1 = n+1 Or f(n)=n Meaning that our solution is f(x) = x Also, when looking at our function, we can easily see that f(x)=0 works. Less rigorous, but similar results.
@jamescollis7650
@jamescollis7650 8 месяцев назад
f has to be injective and surjective to have an inverse
@Reliquancy
@Reliquancy 2 года назад
What type of functions have m*f(x) = f(m*x). Is it just f(x) = a*x? I know linear operators like integrals have that but I guess they aren’t functions.
@pedroteran5885
@pedroteran5885 2 года назад
Yes, because f(x)=f(x*1)=x*f(1), that is, we have a=f(1).
@synaestheziac
@synaestheziac 2 года назад
Does anyone know of, or could anyone create, a functional equation which has both linear and quadratic solutions? I haven’t tried very hard to come up with one but it’s not clear to me how one would do it
@pedroteran5885
@pedroteran5885 2 года назад
There must be smarter or more elegant ones, but consider f(x+1)=f(x)^2+ 2(f(x)-x)/(x-1) +1. It seems to me (it's hard to pay full attention to tedious calculations though) that the only polynomials satisfying that equation are ax^2+bx for a+b=1. I didn't try to find an argument discarding more general functions. Here's how I generated that equation. I started with f(x+1)=x+1, obviously satisfied by f(x)=x. For the square function, we have f(x+1)=f(x)+2x+1, which is rather similar. To cram both identities in one single expression we write f(x+1)=f(x)+1+g(x) and look for some g that becomes 0 for f(x)=x and 2x for f(x)=2x. The first requirement is easy, we'll just put a factor (f(x)-x) in the definition of g to make sure it is 0. That yields x^2-x when f is the square function, while we want 2x. We just multiply by 2x/(x^2-x), that is, 2/(x-1) and we are done: both x and x^2 are solutions to f(x+1)=f(x)+1+ (f(x)-x)*2/(x-1).
@synaestheziac
@synaestheziac 2 года назад
@@pedroteran5885 awesome, thanks!
@avinashthakur80
@avinashthakur80 2 года назад
Non negative integers is called Whole numbers
@teenagekhan2439
@teenagekhan2439 2 года назад
At 13:27 you should require x>a so that m does not equal zero
@crazy4hitman755
@crazy4hitman755 2 года назад
16:03 Is it really correct that f(y)=f(y+b-a), since we have only proved that the function is periodic from some point and not in the beginning as well.
@juliang8676
@juliang8676 2 года назад
Can anyone help explain 9:50? Could the function no just be negative everywhere except its maximum?
@ConManAU
@ConManAU 2 года назад
In the problem statement, the function is defined as having its codomain be the non-negative integers, so f(x) can never be negative.
@lumipakkanen3510
@lumipakkanen3510 2 года назад
It could, but the original equation asks us to only look at functions from the non-negative integers to the non-negative integers.
@juliang8676
@juliang8676 2 года назад
Brill thanks guys
@mtbassini
@mtbassini 2 года назад
So the period o f is infinite?
@physicorum7107
@physicorum7107 2 года назад
Damn , the map tho , lmao , nice problem I think i have seen this one outside of the Indian TST but maybe I am just making stuff up
@lucachiesura5191
@lucachiesura5191 2 года назад
In general q*f(p) = f(q*p), q, p >1, but in general we can extend the function in Z, f( - k) = f(1+f(-k-1)), k>0.
@jackalexander102
@jackalexander102 2 года назад
a q p
@jackalexander102
@jackalexander102 2 года назад
w
@orbitfold
@orbitfold 2 года назад
It's very easy to solve by just setting m=1 -> f(1 + f(n)) = f(1 + n) -> f(n) = n
@ojasdeshpande7296
@ojasdeshpande7296 2 года назад
Bruh
@Cjendjsidj
@Cjendjsidj Год назад
You have to prove that the function is injective if you claim f(n) = n
@3dwardian865
@3dwardian865 2 года назад
You can prove the injectivity from periodicity directly, because periodic implies finite image, but the given assumption for large enough m gives large enough f.
@GreenMeansGOF
@GreenMeansGOF 2 года назад
14:28 How do we know f(b) is the last value in the image?
@khoozu7802
@khoozu7802 2 года назад
x>=a, x=/=0 x=0 satisfies f(0)=0 but not satisfies f(x) =f(x+(b-a)) because it states that f(x) =/=0 for x=/=0
@khoozu7802
@khoozu7802 2 года назад
In other words it means image of f cannot have a repeating value of zero
@khoozu7802
@khoozu7802 2 года назад
However, I still think m=/=0 because m=0 has only one solution that is f(0)=0 where x=0. In this way we can prove f(1), where x=1, x>=a, a=0, or a=1. When a=1, m=x-a=1-1=0 which is not accepted. So, a=0, the repeated value of f(1)=f(1+(b-a)) =f(1+b-0)=f(b+1). Last value of the first non-repeated series=f(b+1-1)=f(b)
@jimschneider799
@jimschneider799 2 года назад
I went a different way at about @1:30. After determining that f(0) = 0 with the substitution m = 0, I determined that f(m^2) = m f(m) with the substitution n = 0, from which it was pretty straightforward to prove that f(n) = C n, for some constant C. Substituting this into the original functional equation gave the only possible values for C of 0 and 1.
@bjornfeuerbacher5514
@bjornfeuerbacher5514 2 года назад
"from which it was pretty straightforward to prove that f(n) = C n, for some constant C" I don't see how this follows straightforward. Could you please explain?
@jimschneider799
@jimschneider799 2 года назад
@@bjornfeuerbacher5514 I was a bit inaccurate in my statement. It's straightforward to prove it, _assuming_ f(n) is a polynomial, which is a bit sloppy. With this assumption, f(n) = a[0] + a[1] n + a[2] n^2 + ... + a[k] n^k. Since f(0) = 0, it must be that a[0] = 0. Expanding the functions on both sides of the equation f(n^2) = n f(n) gives: a[1] n^2 + a[2] n^3 + ... + a[k] n^(k+1) = a[1] n^2 + a[2] n^4 + ... + a[k] n^(2 k) Since these are identically equal, and the expansion on the right hand side has no terms of odd degree, the coefficients of the terms of odd degree on the left hand side must all equal zero, which gives a[2 j] = 0. Equating the remaining terms gives a[2 j-1] = a[j], for all j > 1. When j is even, a[2 j-1] = 0. When it is odd, since j = 2 ((j+1)/2) - 1, j can be replaced by (j+1)/2, giving a[2 j-1] = a[(j+1)/2]. Since (j+1)/2 < j for all j > 1, and there are a finite number of integers between 0 and any finite number j, this descending sequence must eventually terminate. Further, for all odd integers j > 1, the least possible value of (j+1)/2 is 2 (this can be proved by assuming that (j+1)/2
@goodplacetostop2973
@goodplacetostop2973 2 года назад
18:39 Indian flag on the thumbnail gonna bring a lot of viewers and comments…
@maxwellsequation4887
@maxwellsequation4887 2 года назад
Pro tip: Don't use country maps
@kishanpaswan9615
@kishanpaswan9615 2 года назад
You are great...
@sadeekmuhammadryan4894
@sadeekmuhammadryan4894 2 года назад
I don't know if you feel the same or not, but functional equation is the hardest topic for me 😬
@keishakwok4333
@keishakwok4333 6 месяцев назад
No, at this point polynomials bother me more. Hopefully that changes.
@Vladimir_Pavlov
@Vladimir_Pavlov 2 года назад
f(m^2 +m*f(n))=n*f(m+n). (1) Take m=n. f(n^2 +n*f(n))=n*f(2n). (2) Since f(n) represents the set of non-negative integers in the same, then looking for it in the form of a series n with non-negative degrees: f(n)= a0+ a1*n +a2*n^2 +a3*n^3 +... (3) Substituting (3) into (2) seems cumbersome, but it quickly gives the desired result. a0 + a1*(n^2+n*f(n)) +a2*(n^2+n*f(n))^2 +a3 *(n^2 +n*f(n))^3+........= = n*a0+n*a1*2*n+n*a2* (2*n)^2 + n*a3*(2*n)^3+...... a0+a1*n^2 +a1*(n*a0+n*a1*n +n*a2*n^2+n*a3*n^3+....)+ a2*(n^4 +2*n^3 *a0+...+n^2*a0+....=a0*n+2*a1*n^2 +4*a2*n^3+.... It follows that there is a trivial solution: a0= a1=a2=a3...=0, => f(n)≡0 And there is a solution a0=0, a1=1 , a2=a3=...=0. => f(n)=n. (4) Substituting (4) into (1), we get m^2+mn=n*(m+n) => m=n (we take into account that m and n are non-negative integers). Answer: f(n)=n, m=n and f(n)≡0. I note that f(n)=n the solution is not only for m=1, but for m=n.
@fullfungo
@fullfungo 2 года назад
Your step (3) is not well justified. You simply assume that f(.) is a polynomial, rather then prove it.
@aweebthatlovesmath4220
@aweebthatlovesmath4220 Год назад
@@fullfungo he didn't assume that it was a polynomial he assumed it has a series representation which is still not allowed for functions from uncountable sets to uncountable sets but because ℕ∪{0} is countable he can assume there is a series representation...
@ilias-4252
@ilias-4252 2 месяца назад
I got a much better solution but i dont see anyone talking about it so there s a chance it s wrong: We can easily get f(0)=0 , then using the eq f(m)=f(m^2)/m and plugging in x^2 , x^4 etc we get: f(m)=f(m^2)/m=f(m^4)/m^3=....= f(m^(2^n)/m^((2^n)-1). Set u=m^(2^n), assuming m>=2 and sending n to infinity we have : f(m)=m • lim u->inf f(u)/u . The lim has to exist and be a real number (cause it s ewual to f(m) ) so we have f(m)=bm. Plugging back into the original we can easily get b=1, then just find f(1) (cause we assumed m>=2) and we are done
@ilias-4252
@ilias-4252 2 месяца назад
This solution also kinda works even if the domain was the reals
@brabhamfreaman166
@brabhamfreaman166 5 месяцев назад
I know it varies between mathematicians and ‘areas of interest’, but I’d really prefer you remained true and faithful to the Von Neumann (pr/ Vonn Noy-munn) natural numbers which start from Ø (ie. zero, 0), then {Ø}, {Ø,{Ø}}, {Ø, {Ø}, {Ø,{Ø}}} and so on. I’m sure you, Michael, are familiar with Undergraduate Logic & Set Theory, I just include it for completeness and interest for other readers.
@tharunsankar4926
@tharunsankar4926 2 года назад
All this only for f(x) = 0
@juliang8676
@juliang8676 2 года назад
Yeet
@larspos8264
@larspos8264 2 года назад
Just let m=-a at 13:22 and you're instantely done
@dicksonchang6647
@dicksonchang6647 2 года назад
m can not be negative
@larspos8264
@larspos8264 2 года назад
@@dicksonchang6647 why not?
@kimbel2804
@kimbel2804 2 года назад
@@larspos8264 It is in the problem statement that m and n are non-negative.
@larspos8264
@larspos8264 2 года назад
@@kimbel2804 ofcouce, I only saw the Z, my bad
@johanrichter2695
@johanrichter2695 2 года назад
Nice problem. A shame the answer is so often something simple when solving functional equations.
@AcaciaAvenue
@AcaciaAvenue 2 года назад
I'm lost at 9:40. Why do you assume that y and z are integers? It seems to me it's just an hypotesis you made, and as such y+z = 1 can have other solutions, rather than y or z being 1 and the other being 0.
@BerfOfficial
@BerfOfficial 2 года назад
The function f is only defined for integers.
@harshuldesai8901
@harshuldesai8901 2 года назад
f is only defined for Z^nonneg \cup {0} or Z_{\geq 0}, therefore y, z are bound to be either 0 or 1 to have a sum of 1.
@anshumanagrawal346
@anshumanagrawal346 2 года назад
The question states that the domain and co domain of f is the set of non negative integers
Далее
One of the coolest functional equations I have seen!
15:34
A nice functional equation from Romania
18:36
Просмотров 27 тыс.
This problem is everywhere!
17:56
Просмотров 47 тыс.
a very nice Pythagorean triangle puzzle
13:45
Просмотров 16 тыс.
A Fun One from the Vietnamese IMO
13:26
Просмотров 345 тыс.
An integral that is out of this world!!
11:20
a notorious functional equation.
19:30
Просмотров 27 тыс.
Canadian Mathematical Olympiad | 2018 Q2
15:59
Просмотров 72 тыс.
The complex number family.
17:58
Просмотров 69 тыс.
Olympiad level counting  (Generating functions)
34:36
Просмотров 1,9 млн